13.07.2014 Views

November 2001 - Course 1 SOA Solutions

November 2001 - Course 1 SOA Solutions

November 2001 - Course 1 SOA Solutions

SHOW MORE
SHOW LESS

You also want an ePaper? Increase the reach of your titles

YUMPU automatically turns print PDFs into web optimized ePapers that Google loves.

<strong>Course</strong> 1 <strong>Solutions</strong><br />

<strong>November</strong> <strong>2001</strong> Exams


1. A<br />

For i = 1, 2, let<br />

R = event that a red ball is drawn form urn i<br />

i<br />

B = event that a blue ball is drawn from urn i .<br />

i<br />

Then if x is the number of blue balls in urn 2,<br />

( R1∩R2) ∪( B1∩B2) R1∩R2 [ B1∩B2]<br />

[ R ] [ R ] + [ B ] [ B ]<br />

0.44 = Pr[ ] = Pr[ ] + Pr<br />

= Pr Pr Pr Pr<br />

1 2 1 2<br />

4 ⎛ 16 ⎞ 6 ⎛ x ⎞<br />

= ⎜ ⎟+<br />

⎜ ⎟<br />

10 ⎝ x+ 16 ⎠ 10 ⎝ x+<br />

16 ⎠<br />

Therefore,<br />

32 3x<br />

3x+<br />

32<br />

2.2 = + =<br />

x+ 16 x+ 16 x+<br />

16<br />

2.2x+ 35.2 = 3x+<br />

32<br />

0.8x<br />

= 3.2<br />

x = 4<br />

2. C<br />

We are given that<br />

f x, y dA= 6 and dA=<br />

2<br />

∫∫ R<br />

( )<br />

∫∫ R<br />

It follows that<br />

∫∫⎡4 ( , ) 2 4 ( , ) 2<br />

R ⎣ f xy− ⎤⎦dA= ∫∫f xydA−<br />

dA<br />

R ∫∫ R<br />

= 4 6 − 2 2 = 20<br />

( ) ( )<br />

<strong>Course</strong> 1 <strong>Solutions</strong> 1 <strong>November</strong> <strong>2001</strong>


3. A<br />

Since<br />

3 4 2 5<br />

S = 175L A , we have<br />

∂ S<br />

1 4<br />

=<br />

2 5<br />

262.5 L A > 0 for<br />

∂L<br />

L > 0 , A > 0<br />

2<br />

∂ S<br />

− 1 4<br />

2 5<br />

= 131.25 L A > 0 for<br />

2<br />

∂L<br />

L> 0 , A><br />

0<br />

∂ S<br />

3 −1 = 140 L 2<br />

A 5 > 0 for<br />

∂A<br />

L > 0 , A > 0<br />

2<br />

∂ S<br />

3 −6<br />

2 5<br />

=− 28 L A < 0 for<br />

2<br />

∂A<br />

L> 0 , A><br />

0<br />

It follows that S increases at an increasing rate as L increases, while S increases at a<br />

decreasing rate as A increases.<br />

4. B<br />

Apply Baye’s Formula:<br />

Pr ⎡⎣Seri. Surv. ⎤⎦<br />

=<br />

Pr ⎡⎣Surv. Seri. ⎤⎦Pr[ Seri. ]<br />

Pr ⎡⎣Surv. Crit. ⎤ ⎦Pr[ Crit. ] + Pr ⎡⎣Surv. Seri. ⎤ ⎦Pr[ Seri. ] + Pr ⎡⎣Surv. Stab. ⎤⎦Pr[ Stab. ]<br />

( 0.9)( 0.3)<br />

0.29<br />

( 0.6)( 0.1) + ( 0.9)( 0.3) + ( 0.99)( 0.6)<br />

= =<br />

<strong>Course</strong> 1 <strong>Solutions</strong> 2 <strong>November</strong> <strong>2001</strong>


5. A<br />

Let us first determine K. Observe that<br />

⎛ 1 1 1 1 ⎞ ⎛60 + 30 + 20 + 15 + 12 ⎞ ⎛137<br />

⎞<br />

1= K⎜1+ + + + ⎟= K⎜ ⎟=<br />

K⎜ ⎟<br />

⎝ 2 3 4 5 ⎠ ⎝ 60 ⎠ ⎝ 60 ⎠<br />

60<br />

K =<br />

137<br />

It then follows that<br />

Pr N = n = Pr ⎣⎡ N = n Insured Suffers a Loss⎤⎦Pr Insured Suffers a Loss<br />

60 3<br />

= ( 0.05 ) = , N = 1,...,5<br />

137N<br />

137N<br />

P = E X where<br />

[ ] [ ]<br />

Now because of the deductible of 2, the net annual premium [ ]<br />

⎧0 , if N ≤ 2<br />

X = ⎨<br />

⎩N<br />

− 2 , if N > 2<br />

Then,<br />

P E [ X 5<br />

] ( N 3 ⎛ 1 ⎞ ⎡ 3 ⎤ ⎡ 3 ⎤<br />

= = ∑ − 2) = () 1 2 3 0.0314<br />

N = 3<br />

⎜ ⎟+ ⎢ ⎥+ ⎢ ⎥ =<br />

137N<br />

⎝137 ⎠ ⎣137( 4) ⎦ ⎣137( 5)<br />

⎦<br />

6. E<br />

The line 5y− 4x= 3 has slope 4 5<br />

4 dy dy dx 2t<br />

= = =<br />

5 dx dt dt 2t<br />

+ 1<br />

8t+ 4=<br />

10t<br />

2t<br />

= 4<br />

t = 2<br />

. It follows that we need to find t such that<br />

<strong>Course</strong> 1 <strong>Solutions</strong> 3 <strong>November</strong> <strong>2001</strong>


7. A<br />

Cov C , C = Cov X + Y, X + 1.2Y<br />

( 1 2) ( )<br />

= Cov ( X, X) + Cov ( Y, X) + Cov ( X,1.2Y) + Cov( Y,1.2Y)<br />

= Var X + Cov ( X, Y) + 1.2Cov ( X, Y)<br />

+ 1.2VarY<br />

= Var X + 2.2Cov ( X, Y)<br />

+ 1.2VarY<br />

( ) ( ( ))<br />

2<br />

( ) ( E Y )<br />

2 2<br />

2<br />

Var X = E X − E X = 27.4 − 5 = 2.4<br />

VarY<br />

2 2<br />

= E Y − ( ) = 51.4 − 7 = 2.4<br />

( X + Y) = X + Y + ( X Y)<br />

Var Var Var 2Cov ,<br />

1<br />

Cov ( XY , ) = Var ( X+ Y)<br />

−Var X−VarY<br />

2<br />

1<br />

= ( 8 − 2.4 − 2.4 ) = 1.6<br />

2<br />

Cov , = 2.4 + 2.2 1.6 + 1.2 2.4 = 8.8<br />

( C C ) ( ) ( )<br />

1 2<br />

( )<br />

<strong>Course</strong> 1 <strong>Solutions</strong> 4 <strong>November</strong> <strong>2001</strong>


7. Alternate solution:<br />

We are given the following information:<br />

C1<br />

= X + Y<br />

C2<br />

= X + 1.2Y<br />

E X = 5<br />

[ ]<br />

2<br />

⎡ ⎤ =<br />

E⎣X<br />

⎦<br />

EY = 7<br />

[ ]<br />

2<br />

⎡ ⎤ =<br />

27.4<br />

E⎣Y<br />

⎦ 51.4<br />

Var[ X + Y]<br />

= 8<br />

Now we want to calculate<br />

Cov C , C = Cov X + Y, X + 1.2Y<br />

( 1 2) ( )<br />

= E⎡⎣( X + Y)( X + 1.2Y) ⎤⎦− E[ X + Y] iE[ X + 1.2Y]<br />

2 2<br />

= ⎡<br />

⎣ + + ⎤<br />

⎦− [ ] + [ ] +<br />

2 2<br />

= E⎡ ⎣X ⎤<br />

⎦+ 2.2E[ XY] + 1.2E⎡ ⎣Y<br />

⎤<br />

⎦− ( 5 + 7) 5 + ( 1.2)<br />

7<br />

= 27. 4 + 2.2E[ XY] + 1.2( 51.4) −( 12)( 13.4)<br />

= 2.2E[ XY]<br />

−71.72<br />

Therefore, we need to calculate E [ XY ] first. To this end, observe<br />

2<br />

2<br />

8= Var[ X + Y] = E⎡( X + Y) ⎤− ( E[ X + Y]<br />

)<br />

[ ]<br />

E XY<br />

( )( [ ] [ ])<br />

( )<br />

E X 2.2XY 1.2Y E X E Y E X 1.2E Y<br />

⎣<br />

⎦<br />

2 2<br />

2<br />

= E ⎡<br />

⎣X + 2XY + Y ⎤<br />

⎦− ( E[ X] + E[ Y]<br />

)<br />

2 2<br />

2<br />

= E⎡ ⎣X ⎤<br />

⎦+ 2E[ XY] + E⎡ ⎣Y<br />

⎤<br />

⎦− ( 5 + 7)<br />

= 27.4 + 2E[ XY]<br />

+ 51.4 −144<br />

= 2E[ XY]<br />

−65.2<br />

( 8 65.2)<br />

2 36.6<br />

= + =<br />

Finally,<br />

Cov CC = 2.2 36.6 − 71.72 = 8.8<br />

( 1, 2 ) ( )<br />

<strong>Course</strong> 1 <strong>Solutions</strong> 5 <strong>November</strong> <strong>2001</strong>


8. D<br />

The function F() t , 1≤t<br />

≤ 7 , achieves its minimum value at one of the endpoints of the<br />

interval 1≤t<br />

≤ 7 or at t such that<br />

−t −t −t<br />

0= F′<br />

() t = e − te = e ( 1−t)<br />

Since F′ () t < 0 for t > 1 , we see that F () t is a decreasing function on the interval<br />

1< t ≤ 7 . We conclude that<br />

F 7 = 7e − = 0.0064<br />

( )<br />

7<br />

is the minimum value of F.<br />

9. D<br />

Let<br />

C = event that patient visits a chiropractor<br />

T = event that patient visits a physical therapist<br />

We are given that<br />

Pr[ C] = Pr[ T]<br />

+ 0.14<br />

Pr C∩T<br />

= 0.22<br />

( )<br />

c c<br />

( C ∩T<br />

)<br />

Pr = 0.12<br />

Therefore,<br />

c c<br />

0.88 = 1− Pr ⎡<br />

⎣C ∩T ⎤<br />

⎦ = Pr C∪T = Pr C + Pr T −Pr<br />

C∩T<br />

= Pr[ T] + 0.14 + Pr[ T]<br />

−0.22<br />

= 2 Pr T −0.08<br />

or<br />

[ ]<br />

Pr[ T ] = ( 0.88 + 0.08)<br />

2 = 0.48<br />

[ ] [ ] [ ] [ ]<br />

<strong>Course</strong> 1 <strong>Solutions</strong> 6 <strong>November</strong> <strong>2001</strong>


10. E<br />

Note that the terms of ∑ ∞ ⎛ 1 ⎞<br />

a<br />

n=<br />

1⎜<br />

n<br />

+ ⎟<br />

⎝ n ⎠<br />

exceed the terms of the harmonic series for<br />

1 1<br />

a = n<br />

1 , a = n<br />

, or an<br />

2<br />

n<br />

= n<br />

and will thus fail to converge.<br />

Moreover, the terms of the series<br />

n<br />

∞<br />

⎡( −1)<br />

1⎤<br />

∞ ⎛ 1 1 ⎞ ∞ 1<br />

∑ ⎢ + ⎥ =<br />

n= 1 ∑n= 1⎜ + ⎟=<br />

∑n=<br />

1<br />

⎢⎣<br />

n n⎥<br />

⎦ ⎝2n 2n⎠<br />

n<br />

are seen to be identical to the terms of the divergent harmonic series. By contrast, the<br />

series<br />

∞ ⎡ 1⎤ ∞ ⎡1−<br />

n 1⎤<br />

∞ 1<br />

∑ a<br />

n= 1⎢ n<br />

+<br />

n= 1 2 n=<br />

1 2<br />

n⎥ = ∑ + =<br />

⎣ ⎦<br />

⎢<br />

⎣ n n⎥<br />

∑<br />

⎦ n<br />

is seen to converge by the integral test, since<br />

∞<br />

2 1<br />

x −<br />

dx ∞<br />

∫ =−<br />

1 = 1


12. E<br />

Observe that<br />

x<br />

f′ x = f′′<br />

t dt<br />

For [ ]<br />

( ) ( )<br />

∫<br />

0<br />

= (area bounded by f′′<br />

above the x-axis from 0 to x)<br />

− (area bounded by f ′′ below the x-axis from 0 to x)<br />

x ∈ 0,5 , f ′′ is drawn such that the area bounded by f ′′ above the x-axis from 0 to<br />

x is strictly greater than the area bounded by f ′′ below the x-axis from 0 to x. Therefore,<br />

f′ ( x) > 0 for 0 < x≤ 5 . We conclude that f is an increasing function on the interval<br />

[ 0,5 ], and its maximum value occurs at x = 5 .<br />

13. E<br />

10 8<br />

F y Y y X y ⎡ X ⎤ e −<br />

⎣ ⎦ ⎢ 10 ⎥<br />

⎣<br />

⎦<br />

( )<br />

( ) [ ] ( ) 10 8<br />

0.8 10<br />

= Pr ≤ = Pr ⎡10 ≤ ⎤ = Pr ≤ Y = 1−<br />

Therefore, ( ) ′( )<br />

1 4<br />

10<br />

1 Y<br />

f y = F y = ⎛ ⎞<br />

⎜ ⎟ e −<br />

8⎝10⎠<br />

( Y ) 54<br />

Y<br />

14. A<br />

L’Hôspital’s Rule may be applied since f and g are given differentiable and from the<br />

diagram lim f( x) = lim g( x) = 0 .<br />

x→0 x→0<br />

f ( x)<br />

f′<br />

( x)<br />

Therefore lim = lim<br />

x→0 g ( x)<br />

x→0<br />

g′<br />

( x)<br />

Also from the diagram f′ ( x) f′<br />

( )<br />

This leads to<br />

lim = 0 > 0<br />

x→0<br />

and g′ ( x) x→0<br />

g′<br />

( )<br />

f ( x)<br />

f′ ( x)<br />

f′<br />

( 0)<br />

→0 g( x)<br />

x→0<br />

g′ ( x)<br />

g′<br />

( 0)<br />

lim = 0 < 0<br />

lim = lim = < 0<br />

x<br />

<strong>Course</strong> 1 <strong>Solutions</strong> 8 <strong>November</strong> <strong>2001</strong>


15. E<br />

Let X1,...,<br />

X<br />

100<br />

denote the number of pensions that will be provided to each new recruit.<br />

Now under the assumptions given,<br />

⎧0 with probability 1− 0.4 = 0.6<br />

⎪<br />

X i = ⎨1 with probability ( 0.4)( 0.25)<br />

= 0.1<br />

⎪<br />

⎩2 with probability ( 0.4)( 0.75)<br />

= 0.3<br />

for i = 1,...,100 . Therefore,<br />

E X = 0 0.6 + 1 0.1 + 2 0.3 = 0.7 ,<br />

[ i ] ( )( ) ( )( ) ( )( )<br />

2 2 2 2<br />

⎣ i ⎦ ( ) ( ) ( ) ( ) ( ) ( )<br />

2<br />

[ Xi] E⎡X ⎤<br />

i { E[ Xi]<br />

} ( )<br />

E⎡X<br />

⎤ = 0 0.6 + 1 0.1 + 2 0.3 = 1.3 , and<br />

2 2<br />

Var =<br />

⎣ ⎦<br />

− = 1.3 − 0.7 = 0.81<br />

Since X1,...,<br />

X<br />

100<br />

are assumed by the consulting actuary to be independent, the Central<br />

Limit Theorem then implies that S = X1+ ... + X100<br />

is approximately normally distributed<br />

with mean<br />

E S = E X + ... + E X = 100 0.7 = 70<br />

[ ] [ ] [ ] ( )<br />

1 100<br />

and variance<br />

Var S = Var X + ... + Var X = 100 0.81 = 81<br />

[ ] [ ] [ ] ( )<br />

1 100<br />

Consequently,<br />

⎡S<br />

−70 90.5 −70⎤<br />

Pr[ S ≤ 90.5]<br />

= Pr<br />

⎢<br />

≤<br />

⎣ 9 9 ⎥<br />

⎦<br />

= Pr[ Z ≤2.28]<br />

= 0.99<br />

16. C<br />

Observe<br />

Pr 4 < S < 8 = Pr ⎡ ⎣4 < S < 8 N = 1⎤ ⎦Pr N = 1 + Pr ⎡ ⎣4 < S < 8 N > 1⎤ ⎦Pr N > 1<br />

1 − 8<br />

( ) ( )<br />

4 − 1 −<br />

5 5 1 2 −1<br />

= e − e + e −e<br />

*<br />

3 6<br />

= 0.122<br />

*Uses that if X has an exponential distribution with mean µ<br />

[ ] [ ] [ ]<br />

b<br />

1 1<br />

− −<br />

−t<br />

µ −t<br />

µ µ<br />

Pr ( a≤ X ≤ b) = Pr ( X ≥a) −Pr<br />

( X ≥ b)<br />

= ∫ e dt− e dt = e −e µ<br />

µ ∫µ<br />

∞<br />

a<br />

∞<br />

b<br />

a<br />

<strong>Course</strong> 1 <strong>Solutions</strong> 9 <strong>November</strong> <strong>2001</strong>


17. B<br />

Note that<br />

20<br />

[ > ] = ( − )<br />

∫<br />

Pr X x 0.005 20 t dt<br />

x<br />

⎛ 1 2⎞<br />

20<br />

= 0.005⎜20t−<br />

t ⎟ x<br />

⎝ 2 ⎠<br />

⎛<br />

1 2 ⎞<br />

= 0.005⎜400 −200 − 20x<br />

+ x ⎟<br />

⎝<br />

2 ⎠<br />

⎛ 1 2 ⎞<br />

= 0.005⎜200 − 20x+<br />

x ⎟<br />

⎝<br />

2 ⎠<br />

where 0< x < 20 . Therefore,<br />

[ X > ]<br />

[ X ]<br />

( ) 1<br />

2( )<br />

2<br />

( ) 1 ( )<br />

Pr 16 200 − 20 16 + 16 8 1<br />

Pr ⎡ ⎣X<br />

> 16 X > 8⎤ ⎦ = = = =<br />

Pr > 8 200 − 20 8 + 8 72 9<br />

2<br />

2<br />

18. D<br />

Note that<br />

⎧2 x for x< 0 and x><br />

1<br />

⎪<br />

f′ ( x)<br />

= ⎨ 1<br />

⎪<br />

for 0 < x < 1<br />

⎩2<br />

x<br />

Furthermore,<br />

lim<br />

lim<br />

0 = x<br />

− f′ ( x) ≠ ( )<br />

0 x<br />

+ f′<br />

x =+∞<br />

→<br />

→0<br />

and<br />

lim<br />

lim 1<br />

2 = f ( x ) f ( x<br />

+<br />

′ ≠ −<br />

′ ) =<br />

x→1 x→1<br />

2<br />

so f is not differentiable at x = 0 or x = 1, although f is differentiable everywhere else. By<br />

contrast, f is continuous everywhere since<br />

lim<br />

f ( x lim<br />

) f ( x<br />

+ = − ) = 0<br />

x→0 x→0 and<br />

lim<br />

f x lim<br />

= f x =<br />

( ) ( )<br />

+ − 1<br />

x→1 x→1 <strong>Course</strong> 1 <strong>Solutions</strong> 10 <strong>November</strong> <strong>2001</strong>


19. B<br />

The amount of money the insurance company will have to pay is defined by the random<br />

variable<br />

⎧1000 x if x<<br />

2<br />

Y = ⎨<br />

⎩2000 if x ≥ 2<br />

where x is a Poisson random variable with mean 0.6 . The probability function for X is<br />

−0.6<br />

k<br />

e ( 0.6)<br />

p( x)<br />

= k = 0,1, 2,3 $ and<br />

k!<br />

k<br />

−0.6 −0.6<br />

∞ 0.6<br />

EY [ ] = 0 + 1000( 0.6)<br />

e + 2000e<br />

∑ k = 2<br />

k !<br />

k<br />

−0.6 ⎛ −0.6 ∞ 0.6 −0.6 −0.6<br />

⎞<br />

= 1000( 0.6) e + 2000⎜e ∑ −e −( 0.6)<br />

e<br />

k = 0<br />

⎟<br />

⎝ k!<br />

⎠<br />

k<br />

∞<br />

−0.6 ( 0.6)<br />

−0.6 −0.6 −0.6 −0.6<br />

= 2000e ∑ −2000e − 1000( 0.6)<br />

e = 2000 −2000e −600e<br />

k = 0 k!<br />

= 573<br />

k<br />

2 2 −0.6 2 −0.6<br />

∞ 0.6<br />

E⎡ ⎣Y ⎤<br />

⎦ = ( 1000) ( 0.6) e + ( 2000)<br />

e ∑ k = 2<br />

k!<br />

k<br />

2 −0.6 ∞ 0.6<br />

2 −0.6 2 2<br />

−0.6<br />

= ( 2000) e ∑ −( 2000) e −⎡( 2000) −( 1000) ⎤( 0.6)<br />

e<br />

k = 0<br />

k!<br />

⎣<br />

⎦<br />

2 2 −0.6<br />

2 2<br />

= ( 2000) −( 2000)<br />

e −⎡<br />

−0.6<br />

( 2000) − ( 1000) ⎤( 0.6)<br />

e<br />

⎣<br />

⎦<br />

= 816,893<br />

Var<br />

2<br />

[ Y] = E⎡<br />

⎣Y ⎤<br />

⎦−{ E[ Y]<br />

}<br />

2<br />

= 816,893−( 573)<br />

= 488,564<br />

[ Y ]<br />

Var = 699<br />

2<br />

20. B<br />

The graph of E′ tells us that employment increases from April through August (because<br />

E t<br />

E′ t ≤ then).<br />

′() > 0 then) and does not increase from August through April (since () 0<br />

It follows that employment is a minimum in April.<br />

<strong>Course</strong> 1 <strong>Solutions</strong> 11 <strong>November</strong> <strong>2001</strong>


21. D<br />

Let N1 and N<br />

2<br />

denote the number of claims during weeks one and two, respectively.<br />

Then since N1 and N<br />

2<br />

are independent,<br />

7<br />

[ N1+ N2 = ] = ∑ [ N1 = n] [ N2<br />

= −n]<br />

n=<br />

0<br />

Pr 7 Pr Pr 7<br />

7 ⎛ 1 ⎞⎛ 1 ⎞<br />

= ∑n=<br />

0⎜ n+ 1⎟⎜ 8−n<br />

⎟<br />

⎝2 ⎠⎝2<br />

⎠<br />

7 1<br />

= ∑n=<br />

0 9<br />

2<br />

8 1 1<br />

= = =<br />

9 6<br />

2 2 64<br />

22. D<br />

The amount of the drug present peaks at the instants an injection is administered. If A(n)<br />

is the amount of the drug remaining in the patient’s body at the time of the nth injection,<br />

then<br />

A 1 = 250<br />

()<br />

−16<br />

( 2) = 250( 1+<br />

e )<br />

A<br />

%<br />

n−1 −k<br />

6<br />

An ( ) = 250∑<br />

e<br />

k = 0<br />

and the least upper bound is<br />

∞ −k<br />

6 250<br />

lim An ( ) = 250∑<br />

e = = 1628 (noting the<br />

n→∞<br />

k = 0<br />

−16<br />

1−<br />

e<br />

−16<br />

series is geometric with ratio e )<br />

<strong>Course</strong> 1 <strong>Solutions</strong> 12 <strong>November</strong> <strong>2001</strong>


23. C<br />

Observe that<br />

sin ( θ + π) + 3 cos ( θ + π)<br />

=− ⎡<br />

⎣<br />

sinθ + 3 cosθ⎤<br />

⎦<br />

r, θ and − r,<br />

θ + π define the same point, and we may restrict attention to<br />

Therefore, ( ) ( )<br />

π<br />

θ such that 0 ≤ θ < π . Now for 0 ≤θ<br />

≤ , 0<br />

2 r > and the graph of r is in the first<br />

π<br />

π<br />

quadrant (i.e., to the right of θ = ). On the other hand, for < θ < π, r > 0 when<br />

2<br />

2<br />

sinθ<br />

+ 3 cosθ<br />

> 0<br />

sinθ<br />

> 3 cosθ<br />

sinθ<br />

> 3<br />

cosθ<br />

tanθ<br />

> 3<br />

⎡π<br />

⎤<br />

This is true in the interval<br />

⎢<br />

, π<br />

⎣ 2 ⎥<br />

⎦ when π 2π<br />

< θ < . Consequently, the area bounded<br />

2 3<br />

π<br />

2π<br />

3<br />

2<br />

by the graph of r to the left of θ = is given by rd<br />

2<br />

∫ θ .<br />

π 2<br />

[It is not needed to do the problem, but the graph of the curve is a circle of radius 1<br />

3 1<br />

centered at x= , y = ].<br />

2 2<br />

<strong>Course</strong> 1 <strong>Solutions</strong> 13 <strong>November</strong> <strong>2001</strong>


24. D<br />

1 1<br />

4 2<br />

100 x y must be maximized subject to x+ y = 150,000<br />

Since y = 150,000 −x<br />

, this reduces to maximizing<br />

( ) ( )<br />

1 1<br />

4 2<br />

S x = 100x 150,000 −x , 0 ≤ x≤150,000<br />

−3 1<br />

4 4<br />

S ( x) x ( x) x ( x)<br />

( 150,000 −x)<br />

− 2x=<br />

0<br />

1 2 −1<br />

2<br />

′ = 25 150,000 − −50 150,000 − = 0<br />

3x<br />

= 150,000<br />

x = 50, 000<br />

(This value of x is a maximum since S ( x) 0<br />

50,000 < x < 150,000 ).<br />

′ > for 0 x 50,000<br />

1 1<br />

4 2<br />

Maximum sales are then ( ) ( )<br />

Alternate solution using Lagrange Multipliers<br />

Solve x+ y− 150,000 = 0<br />

∂ 1 1 ∂<br />

4 2<br />

100x y = λ x+ y−150,000<br />

∂x<br />

∂x<br />

∂ 1 1 ∂<br />

4 2<br />

100x y = λ x+ y−150,000<br />

∂y<br />

∂y<br />

From the last two equations<br />

25x<br />

50x<br />

Eliminating λ<br />

−3 1<br />

4 2<br />

1 4<br />

y<br />

y<br />

−1 2<br />

′ < for<br />

< < and S ( x) 0<br />

100 50,000 150,000 − 50,000 = 472,871<br />

= λ<br />

= λ<br />

−3 1 1 −1<br />

4 2 4 2<br />

25x y = 50x y<br />

25y<br />

= 50x<br />

y = 2x<br />

Using the first equation<br />

x+ 2x− 150,000 = 0<br />

x = 50,000<br />

y = 100,000<br />

( )<br />

( )<br />

1 1<br />

4 2<br />

( ) ( )<br />

The extreme value (which must be a maximum) is 100 50,000 100,000 = 472,871<br />

<strong>Course</strong> 1 <strong>Solutions</strong> 14 <strong>November</strong> <strong>2001</strong>


25. E<br />

3 f<br />

Pr ⎣⎡ 1< Y < 3 X = 2⎤ ⎦ = ∫ dy<br />

1<br />

f<br />

1<br />

( − )<br />

( 2, y)<br />

( 2)<br />

2 ( ) 1<br />

f ( 2, y)<br />

= y = y<br />

4 2 1 2<br />

∞<br />

− 4−1 2−1 −3<br />

∞<br />

−3 −2<br />

1 1 1<br />

fx<br />

( 2)<br />

= ∫ y dy =− y =<br />

2 4 4<br />

x<br />

1<br />

1<br />

3<br />

−3<br />

y dy<br />

1 2<br />

3<br />

2<br />

− 1 8<br />

1 1<br />

9 9<br />

Finally, Pr ⎡ ⎣1 < Y < 3 X = 2⎤ ⎦ = =− y = 1− =<br />

∫<br />

4<br />

26. C<br />

The increase in sales from year two to year four that is attributable to the advertising<br />

campaign is given by<br />

4⎡⎛ 4<br />

2 1⎞ ⎛ 5⎞⎤<br />

2<br />

∫<br />

( 2)<br />

2 ⎢⎜t + ⎟− ⎜t + ⎟<br />

2 2<br />

⎥dt = ∫ t −t − dt<br />

2<br />

⎣⎝ ⎠ ⎝ ⎠⎦<br />

3 2<br />

⎛t<br />

t ⎞ 4<br />

= ⎜ − −2t<br />

⎟ 2<br />

⎝ 3 2 ⎠<br />

64 8<br />

= −8 −8 − + 2 + 4<br />

3 3<br />

56 26<br />

= − 10 =<br />

3 3<br />

<strong>Course</strong> 1 <strong>Solutions</strong> 15 <strong>November</strong> <strong>2001</strong>


27. D<br />

Let X denote the number of employees that achieve the high performance level. Then X<br />

follows a binomial distribution with parameters n= 20 and p = 0.02 . Now we want to<br />

determine x such that<br />

Pr X > x ≤0.01<br />

[ ]<br />

or, equivalently,<br />

20<br />

0.99 Pr[ ] x ( )( 0.02) k ( 0.98) 20 −<br />

≤ X ≤ x =∑ k<br />

k = 0 k<br />

The following table summarizes the selection process for x:<br />

x Pr X = x Pr X ≤ x<br />

[ ] [ ]<br />

20<br />

( ) =<br />

19<br />

( )( ) =<br />

2 18<br />

( ) ( ) = 53 0.993<br />

0 0.98 0.668 0.668<br />

1 20 0.02 0.98 0.272 0.940<br />

2 190 0.02 0.98 0.0<br />

Consequently, there is less than a 1% chance that more than two employees will achieve<br />

the high performance level. We conclude that we should choose the payment amount C<br />

such that<br />

2C = 120,000<br />

or<br />

C = 60,000<br />

<strong>Course</strong> 1 <strong>Solutions</strong> 16 <strong>November</strong> <strong>2001</strong>


28. B<br />

Let X and Y denote the two bids. Then the graph below illustrates the region over which<br />

X and Y differ by less than 20:<br />

Based on the graph and the uniform distribution:<br />

1<br />

− ⋅<br />

Shaded Region Area<br />

Pr 20<br />

2<br />

⎣⎡ X − Y < ⎤ ⎦ = =<br />

2 2<br />

200<br />

( 2200 − 2000)<br />

( )<br />

2<br />

200 2 180<br />

2<br />

180<br />

2<br />

= 1− = 1− 2<br />

( 0.9)<br />

= 0.19<br />

200<br />

More formally (still using symmetry)<br />

Pr ⎡⎣ X − Y < 20⎤ ⎦ = 1−Pr ⎡⎣ X −Y ≥20⎤ ⎦ = 1−2Pr[ X −Y<br />

≥20]<br />

2200 x−20 1 2200 1 x−20<br />

= 1− 2∫ dydx 1 2 y<br />

2 2 2000<br />

dx<br />

2020 ∫ = −<br />

2000<br />

200<br />

∫2020<br />

200<br />

2 1<br />

= 1− x−20 − 2000 dx= 1− x−2020<br />

2 2020<br />

2<br />

200<br />

∫<br />

200<br />

⎛180<br />

⎞<br />

= 1− ⎜ ⎟ = 0.19<br />

⎝200<br />

⎠<br />

( ) ( )<br />

2200 2 2200<br />

2020<br />

2<br />

2<br />

<strong>Course</strong> 1 <strong>Solutions</strong> 17 <strong>November</strong> <strong>2001</strong>


29. B<br />

Let X and Y denote repair cost and insurance payment, respectively, in the event the auto<br />

is damaged. Then<br />

⎧0 if x ≤ 250<br />

Y = ⎨<br />

⎩x<br />

− 250 if x > 250<br />

and<br />

2<br />

1500 1 1 2<br />

[ ] ( ) ( )<br />

1500 1250<br />

EY = ∫ x− 250 dx= x− 250<br />

250<br />

= = 521<br />

250<br />

1500 3000 3000<br />

3<br />

1500<br />

2 1 2 1 3 1500 1250<br />

E⎡ ⎣Y ⎤<br />

⎦ = ∫ ( x− 250) dx= ( x− 250)<br />

250<br />

= = 434,028<br />

250<br />

1500 4500 4500<br />

2<br />

2 2<br />

Var[ Y] = E⎡<br />

⎣Y ⎤<br />

⎦− { E[ Y]<br />

} = 434,028 −( 521)<br />

Var Y = 403<br />

[ ]<br />

<strong>Course</strong> 1 <strong>Solutions</strong> 18 <strong>November</strong> <strong>2001</strong>


30. C<br />

Let ( , )<br />

below:<br />

and<br />

f t1 t<br />

2<br />

denote the joint density function of<br />

1 2<br />

T<br />

T . The domain of f is pictured<br />

Now the area of this domain is given by<br />

A = 6 −<br />

2<br />

6− 4 = 36− 2=<br />

34<br />

Consequently,<br />

⎧ 1<br />

⎪ , 0< t1 < 6 , 0< t2 < 6 , t1+ t2<br />

< 10<br />

f ( t1, t2)<br />

= ⎨34<br />

⎪⎩ 0 elsewhere<br />

and<br />

2 1 ( ) 2<br />

[ + ] = [ ] + [ ] = [ ]<br />

ET1 T2 ET1 ET2 2 ET1<br />

(due to symmetry)<br />

⎧ 4 6 1 6 10−t1<br />

1 ⎫<br />

= 2⎨∫ t1 2 1 1 2 1<br />

0 ∫ dt dt +<br />

0<br />

34<br />

∫ t<br />

4 ∫ dt dt ⎬<br />

0<br />

⎩<br />

34 ⎭<br />

⎧ 4 ⎡t<br />

6<br />

2 6 ⎤ ⎡t2<br />

10−t<br />

⎤ ⎫<br />

1<br />

= 2⎨∫<br />

t1 0 1 1 0 1<br />

0 ⎢34 ⎥<br />

dt + ∫ t<br />

4 ⎢34<br />

⎥<br />

dt ⎬<br />

⎩ ⎣ ⎦ ⎣ ⎦ ⎭<br />

⎧ 43t 6<br />

1<br />

1<br />

2 ⎫<br />

= 2⎨∫<br />

dt<br />

1+ ( 10 t<br />

1 t<br />

1 ) dt<br />

1<br />

0<br />

17<br />

∫ − ⎬<br />

4<br />

⎩<br />

34<br />

⎭<br />

2<br />

⎧3t1<br />

4 1 ⎛ 2 1 3⎞<br />

6<br />

⎫<br />

= 2⎨<br />

0+ ⎜5t1 − t1<br />

⎟ 4 ⎬<br />

⎩34 34 ⎝ 3 ⎠ ⎭<br />

⎧24 1 ⎡<br />

64⎤⎫<br />

= 2 ⎨ + 180 72 80<br />

17 34 ⎢<br />

− − + ⎬<br />

3 ⎥<br />

⎩ ⎣<br />

⎦⎭<br />

= 5.7<br />

<strong>Course</strong> 1 <strong>Solutions</strong> 19 <strong>November</strong> <strong>2001</strong>


31. E<br />

The general solution of the differential equation may be determined as follows:<br />

1<br />

∫ dy = ( k1 k2)<br />

dt<br />

y<br />

∫ −<br />

ln y = k − k t+<br />

C<br />

(C is a constant)<br />

( ) ( 1 2)<br />

c ( k1 k2)<br />

t<br />

() = e e −<br />

y t<br />

When t = 0 ,<br />

c<br />

y( 0)<br />

= e , so<br />

() ( ) ( k ) 1 k2<br />

t<br />

y t = y 0 e −<br />

1<br />

Now we are given that if k1<br />

= 0, y( 8) = y( 0)<br />

.<br />

2<br />

Therefore,<br />

1<br />

−8k2<br />

y( 0) = y( 8) = y( 0)<br />

e<br />

2<br />

1 −8k2<br />

= e<br />

2<br />

8k2<br />

= ln( 2)<br />

1<br />

k2<br />

= ln ( 2 )<br />

8<br />

24( k1−k2)<br />

[Note the problem also gives y( 24) = 2y( 0) = y( 0)<br />

e , but that information is not<br />

needed to determine k<br />

2<br />

].<br />

32. B<br />

Observe<br />

[ N ]<br />

[ N ]<br />

Pr 1 ≤ ≤4 ⎡1 1 1 1 ⎤ ⎡1 1 1 1 1 ⎤<br />

Pr ⎡⎣N<br />

≥1 N ≤4⎤ ⎦ = = + + + + + + +<br />

Pr ≤ 4 ⎢<br />

⎣6 12 20 30⎥ ⎦<br />

⎢<br />

⎣2 6 12 20 30⎥<br />

⎦<br />

10 + 5 + 3 + 2 20 2<br />

= = =<br />

30 + 10 + 5 + 3 + 2 50 5<br />

<strong>Course</strong> 1 <strong>Solutions</strong> 20 <strong>November</strong> <strong>2001</strong>


33. E<br />

Let X and Y denote the year the device fails and the benefit amount, respectively. Then<br />

the density function of X is given by<br />

x−1<br />

f ( x) = ( 0.6) ( 0.4 ) , x=<br />

1,2,3...<br />

and<br />

⎧⎪ 1000( 5 − x)<br />

if x=<br />

1,2,3, 4<br />

⎪⎩<br />

It follows that<br />

y = ⎨<br />

0 if x > 4<br />

2 3<br />

[ ] 4000( 0.4) 3000( 0.6)( 0.4) 2000( 0.6) ( 0.4) 1000( 0.6) ( 0.4)<br />

EY = + + +<br />

= 2694<br />

34. D<br />

The diagram below illustrates the domain of the joint density ( )<br />

f xy , of Xand<br />

Y.<br />

We are told that the marginal density function of X is f ( x) = 1,0< x<<br />

1<br />

while f ( yx) = 1, x< y< x+<br />

1<br />

yx<br />

It follows that f ( x,<br />

y) f ( x) f ( y x)<br />

Therefore,<br />

⎧1 if 0< x < 1, x< y< x+<br />

1<br />

= x yx<br />

=⎨<br />

⎩ 0 otherwise<br />

[ ] [ ]<br />

1 1<br />

2 2<br />

0 ∫ x<br />

Pr Y > 0.5 = 1−Pr Y ≤ 0.5 = 1−<br />

dydx<br />

∫<br />

1 1<br />

1<br />

2 2<br />

2⎛1<br />

⎞<br />

= 1− ∫ y<br />

x<br />

dx= 1− 0 ∫0<br />

⎜ −x⎟dx<br />

⎝2<br />

⎠<br />

⎡1 1 1<br />

2 ⎤ 1 1 7<br />

2<br />

= 1− ⎢<br />

x− x<br />

0<br />

= 1− + =<br />

⎣2 2 ⎥<br />

⎦ 4 8 8<br />

[Note since the density is constant over the shaded parallelogram in the figure the<br />

solution is also obtained as the ratio of the area of the portion of the parallelogram above<br />

y = 0.5 to the entire shaded area.]<br />

x<br />

<strong>Course</strong> 1 <strong>Solutions</strong> 21 <strong>November</strong> <strong>2001</strong>


35. E<br />

If X is the random variable representing claim amounts, the probability that X exceeds the<br />

deductible is<br />

∞<br />

[ ] ∫<br />

1<br />

−x −x ∞ −x<br />

Pr X > 1 = xe dx=− xe + e dx (integration by parts)<br />

1 1<br />

−1 −x<br />

∞ −1 −1 −1<br />

1<br />

2<br />

= e − e = e + e = e<br />

= 0.736<br />

It follows that the company expects ( 100)( 0.736)<br />

= 74 claims.<br />

∫<br />

∞<br />

36. C<br />

Let<br />

Then<br />

x = price in excess of 60 that the company charges,<br />

p(x) = price per policy that the company charges,<br />

n(x) = number of policies the company sells per month, and<br />

R(x) = revenue per month the company collects<br />

( ) = 60 + x<br />

( ) = 80 −x<br />

( ) = ( ) ( ) = ( 80 − )( 60 + )<br />

′( ) =− ( 60 + ) + ( 80 − ) = 20 −2<br />

′′( x)<br />

=− 2<<br />

0<br />

p x<br />

n x<br />

R x n x p x x x<br />

R x x x x<br />

R<br />

It follows that R(x) is a maximum when R ( x) 0<br />

20 − 2x<br />

= 0<br />

x = 10<br />

R 10 = 80 − 10 60 + 10 = 4900<br />

and ( ) ( )( )<br />

when R(x) is a maximum.<br />

′ = . We conclude that<br />

<strong>Course</strong> 1 <strong>Solutions</strong> 22 <strong>November</strong> <strong>2001</strong>


37. A<br />

The joint density of T1 and T<br />

2<br />

is given by<br />

−t<br />

−t<br />

f t , t = e e , t > 0 , t > 0<br />

( )<br />

1 2<br />

1 2 1 2<br />

Therefore,<br />

Pr X ≤ x = Pr 2T + T ≤ x<br />

[ ] [ ]<br />

1 2<br />

⎡<br />

= ∫∫<br />

= −<br />

⎢⎣<br />

1<br />

1<br />

x ( x−t2<br />

) x<br />

( x−t2<br />

)<br />

2 −t1 −t2 −t2 −t1<br />

2<br />

e e dt1dt2 e e dt2<br />

0 0 ∫ ⎢ ⎥<br />

0<br />

0<br />

⎡ ⎤ ⎛ ⎞<br />

= ∫ − = −<br />

⎣ ⎦ ⎝ ⎠<br />

1 1 1 1<br />

x<br />

− x+ t2 x<br />

− x − t<br />

−t<br />

2<br />

2 2 2 −t2<br />

2 2<br />

e ⎢1<br />

e ⎥dt2 ⎜e e e ⎟dt2<br />

0 ∫0<br />

⎡<br />

⎤<br />

= ⎢− e + e e ⎥ =− e + e e + − e<br />

⎣<br />

⎦<br />

1 1 1 1 1<br />

− x − t2<br />

− x − x − x<br />

−t2<br />

2 2 x − x 2 2 2<br />

2<br />

0<br />

2 1 2<br />

1 1<br />

− x<br />

− x<br />

− x −x<br />

2 2 −x<br />

= 1− e + 2e − 2e = 1− 2 e + e , x><br />

0<br />

It follows that the density of X is given by<br />

1<br />

d ⎡ − x<br />

2 − x<br />

⎤<br />

g ( x)<br />

= ⎢1− 2e + e ⎥<br />

dx ⎣<br />

⎦<br />

1<br />

− x<br />

2<br />

− x<br />

= e − e , x><br />

0<br />

⎤<br />

⎥⎦<br />

38. E<br />

X , X , and X denote annual loss due to storm, fire, and theft, respectively. In<br />

Let<br />

1 2 3<br />

addition, let Y Max( X , X , X )<br />

Then<br />

= .<br />

1 2 3<br />

[ Y > ] = − [ Y ≤ ] = − [ X ≤ ] [ X ≤ ] [ X ≤ ]<br />

Pr 3 1 Pr 3 1 Pr 3 Pr 3 Pr 3<br />

(<br />

−<br />

e )( −3 e )( −3<br />

e )<br />

(<br />

−<br />

e 3 )(<br />

−<br />

e )( −5<br />

e )<br />

= 1− 1− 1− 1−<br />

= 1− 1− 1− 1−<br />

1 2 3<br />

= 0.414<br />

* Uses that if X has an exponential distribution with mean µ<br />

1<br />

Pr ( X ≤ x) = 1−Pr ( X ≥ x) = 1− ∫ e dt = 1−( − e ) x<br />

= 1−e<br />

µ<br />

∞<br />

x<br />

*<br />

−t µ −t µ ∞ −x<br />

µ<br />

<strong>Course</strong> 1 <strong>Solutions</strong> 23 <strong>November</strong> <strong>2001</strong>


39. D<br />

We are given that<br />

P x =− x + 50x−25<br />

( )<br />

2<br />

before the translation occurs. The revised profit function P ( x)<br />

follows:<br />

P*<br />

x = P x−2 −3<br />

( ) ( )<br />

2<br />

( x ) ( x )<br />

=− − 2 + 50 −2 −25−3<br />

2<br />

=− x + x− + x− −<br />

2<br />

=− x + x−<br />

4 4 50 100 28<br />

54 132<br />

* may be determined as<br />

40. B<br />

Observe that<br />

E X + Y = E X + E Y = 50 + 20 = 70<br />

[ ] [ ] [ ]<br />

[ ] [ ] [ ] [ ]<br />

Var X + Y = Var X + Var Y + 2 Cov X , Y = 50 + 30 + 20 = 100<br />

for a randomly selected person. It then follows from the Central Limit Theorem that T is<br />

approximately normal with mean<br />

ET = 100 70 = 7000<br />

[ ] ( )<br />

and variance<br />

Var T = 100 100 = 100<br />

[ ] ( )<br />

2<br />

Therefore,<br />

⎡T<br />

−7000 7100 −7000⎤<br />

Pr[ T < 7100]<br />

= Pr<br />

⎢<br />

<<br />

⎣ 100 100 ⎥<br />

⎦<br />

= Pr[ Z < 1]<br />

= 0.8413<br />

where Z is a standard normal random variable.<br />

<strong>Course</strong> 1 <strong>Solutions</strong> 24 <strong>November</strong> <strong>2001</strong>

Hooray! Your file is uploaded and ready to be published.

Saved successfully!

Ooh no, something went wrong!